www.vorhilfe.de
- Förderverein -
Der Förderverein.

Gemeinnütziger Verein zur Finanzierung des Projekts Vorhilfe.de.
Hallo Gast!einloggen | registrieren ]
Startseite · Mitglieder · Impressum
Forenbaum
^ Forenbaum
Status VH e.V.
  Status Vereinsforum

Gezeigt werden alle Foren bis zur Tiefe 2

Navigation
 Startseite...
 Suchen
 Impressum
Das Projekt
Server und Internetanbindung werden durch Spenden finanziert.
Organisiert wird das Projekt von unserem Koordinatorenteam.
Hunderte Mitglieder helfen ehrenamtlich in unseren moderierten Foren.
Anbieter der Seite ist der gemeinnützige Verein "Vorhilfe.de e.V.".
Partnerseiten
Weitere Fächer:

Open Source FunktionenplotterFunkyPlot: Kostenloser und quelloffener Funktionenplotter für Linux und andere Betriebssysteme
Forum "Topologie und Geometrie" - Wegzusammenhang
Wegzusammenhang < Topologie+Geometrie < Hochschule < Mathe < Vorhilfe
Ansicht: [ geschachtelt ] | ^ Forum "Topologie und Geometrie"  | ^^ Alle Foren  | ^ Forenbaum  | Materialien

Wegzusammenhang: Korrektur
Status: (Frage) beantwortet Status 
Datum: 11:22 Sa 26.05.2012
Autor: kullinarisch

Aufgabe
Sei X ein topologischer Raum, und sei [mm] (M_k)_{k\in\IN} [/mm] eine Familie von Teilmengen von X mit der Eigenschaft [mm] M_k\cap M_{k+1}\not=\emptyset [/mm] für alle [mm] k\in\IN. [/mm] Zeige:

Ist jedes [mm] M_k [/mm] wegzusammenhängend, so auch [mm] \bigcup_{k\in\IN}^{}M_k [/mm]

Hallo zusammen. Wenn ich [mm] x, y \in\bigcup_{k\in\IN}^{}M_k [/mm] beliebig wähle, reicht es doch, wenn ich festlege, dass [mm] x\in M_i [/mm] und [mm] y\in M_{i+1} [/mm] und ich dann zeige, dass [mm] M_i \cup M_{i+1} [/mm] für ein [mm] i\in\IN [/mm] wegzusammenhängend ist. Also ein Punkt an dem man "O.B.d.A" schreiben darf, oder? Ich nehme mal an ja und mache dann so weiter:

Nach Vor. ist [mm] M_i\cap M_{i+1}\not=\emptyset [/mm] also gibt es ein [mm] z\in M_i\cap M_{i+1}. [/mm]

Da beide Mengen wegzusammenhängend sind gibt es 2 stetige Abbildungen:

[mm] \delta_1: [a_1, a_2]\to [/mm] X [mm] \delta_1(a_1)=x, \delta_1(a_2)=z [/mm] in [mm] M_i [/mm]

[mm] \delta_2: [a_2, a_3]\to [/mm] X [mm] \delta_2(a_2)=z, \delta_2(a_3)=y [/mm] in [mm] M_{i+1} [/mm]

Dann ist die Abbildung:

[mm] \alpha: [a_1, a_2]\cup[a_2, a_3]\to [/mm] X mit [mm] \alpha(p)=:\begin{cases} \delta_1(p), & \mbox{für } p\in[a_1, a_2) \\ \delta_2(p), & \mbox{für } p\in[a_2, a_3]\end{cases} [/mm]

stetig.

Ist es so offensichtlich, dass [mm] \alpha [/mm] stetig ist, oder sollte ich das noch zeigen?
Ich habe das ja jetzt nur für 2 "benachbarte" Mengen gezeigt.. ist es denn richtig zu folgern, dass [mm] \bigcup_{k\in\IN}^{}M_k [/mm] wegzusammenhängend ist? Weil ich am Anfang O.B.d.A geschrieben habe?

Mfg, kullinarisch

        
Bezug
Wegzusammenhang: Antwort
Status: (Antwort) fertig Status 
Datum: 12:01 Sa 26.05.2012
Autor: donquijote


> Sei X ein topologischer Raum, und sei [mm](M_k)_{k\in\IN}[/mm] eine
> Familie von Teilmengen von X mit der Eigenschaft [mm]M_k\cap M_{k+1}\not=\emptyset[/mm]
> für alle [mm]k\in\IN.[/mm] Zeige:
>  
> Ist jedes [mm]M_k[/mm] wegzusammenhängend, so auch
> [mm]\bigcup_{k\in\IN}^{}M_k[/mm]
>  Hallo zusammen. Wenn ich [mm]x, y \in\bigcup_{k\in\IN}^{}M_k[/mm]
> beliebig wähle, reicht es doch, wenn ich festlege, dass
> [mm]x\in M_i[/mm] und [mm]y\in M_{i+1}[/mm] und ich dann zeige, dass [mm]M_i \cup M_{i+1}[/mm]
> für ein [mm]i\in\IN[/mm] wegzusammenhängend ist. Also ein Punkt an
> dem man "O.B.d.A" schreiben darf, oder? Ich nehme mal an ja
> und mache dann so weiter:
>  
> Nach Vor. ist [mm]M_i\cap M_{i+1}\not=\emptyset[/mm] also gibt es
> ein [mm]z\in M_i\cap M_{i+1}.[/mm]
>  
> Da beide Mengen wegzusammenhängend sind gibt es 2 stetige
> Abbildungen:
>  
> [mm]\delta_1: [a_1, a_2]\to[/mm] X [mm]\delta_1(a_1)=x, \delta_1(a_2)=z[/mm]
> in [mm]M_i[/mm]
>  
> [mm]\delta_2: [a_2, a_3]\to[/mm] X [mm]\delta_2(a_2)=z, \delta_2(a_3)=y[/mm]
> in [mm]M_{i+1}[/mm]
>  
> Dann ist die Abbildung:
>  
> [mm]\alpha: [a_1, a_2]\cup[a_2, a_3]\to[/mm] X mit
> [mm]\alpha(p)=:\begin{cases} \delta_1(p), & \mbox{für } p\in[a_1, a_2) \\ \delta_2(p), & \mbox{für } p\in[a_2, a_3]\end{cases}[/mm]
>  
> stetig.
>  
> Ist es so offensichtlich, dass [mm]\alpha[/mm] stetig ist, oder
> sollte ich das noch zeigen?

Man könnte noch kurz begründen, warum [mm] \alpha [/mm] in [mm] p=a_2 [/mm] stetig ist

> Ich habe das ja jetzt nur für 2 "benachbarte" Mengen
> gezeigt.. ist es denn richtig zu folgern, dass
> [mm]\bigcup_{k\in\IN}^{}M_k[/mm] wegzusammenhängend ist?

Auch das solltest du noch näher begründen:
Durch Wiederholung des gleichen Argumentes (formal durch Induktion nach n) folgt, dass [mm] A_n=\bigcup_{k=1}^nM_k [/mm] für jedes n wegzusammenhängend ist.
Für den Wegzusammenhang der unendlichen Vereinigung brauchst du dann dein Argument vom Anfang:
Sind zwei Punkte x,y gegeben, so gibt es ein n mit [mm] x,y\in A_n [/mm] und damit einen Weg zwischen x und y.

> Weil ich
> am Anfang O.B.d.A geschrieben habe?
>  
> Mfg, kullinarisch


Bezug
                
Bezug
Wegzusammenhang: Mitteilung
Status: (Mitteilung) Reaktion unnötig Status 
Datum: 14:28 So 27.05.2012
Autor: kullinarisch


> > Sei X ein topologischer Raum, und sei [mm](M_k)_{k\in\IN}[/mm] eine
> > Familie von Teilmengen von X mit der Eigenschaft [mm]M_k\cap M_{k+1}\not=\emptyset[/mm]
> > für alle [mm]k\in\IN.[/mm] Zeige:
>  >  
> > Ist jedes [mm]M_k[/mm] wegzusammenhängend, so auch
> > [mm]\bigcup_{k\in\IN}^{}M_k[/mm]
>  >  Hallo zusammen. Wenn ich [mm]x, y \in\bigcup_{k\in\IN}^{}M_k[/mm]
> > beliebig wähle, reicht es doch, wenn ich festlege, dass
> > [mm]x\in M_i[/mm] und [mm]y\in M_{i+1}[/mm] und ich dann zeige, dass [mm]M_i \cup M_{i+1}[/mm]
> > für ein [mm]i\in\IN[/mm] wegzusammenhängend ist. Also ein Punkt an
> > dem man "O.B.d.A" schreiben darf, oder? Ich nehme mal an ja
> > und mache dann so weiter:
>  >  
> > Nach Vor. ist [mm]M_i\cap M_{i+1}\not=\emptyset[/mm] also gibt es
> > ein [mm]z\in M_i\cap M_{i+1}.[/mm]
>  >  
> > Da beide Mengen wegzusammenhängend sind gibt es 2 stetige
> > Abbildungen:
>  >  
> > [mm]\delta_1: [a_1, a_2]\to[/mm] X [mm]\delta_1(a_1)=x, \delta_1(a_2)=z[/mm]
> > in [mm]M_i[/mm]
>  >  
> > [mm]\delta_2: [a_2, a_3]\to[/mm] X [mm]\delta_2(a_2)=z, \delta_2(a_3)=y[/mm]
> > in [mm]M_{i+1}[/mm]
>  >  
> > Dann ist die Abbildung:
>  >  
> > [mm]\alpha: [a_1, a_2]\cup[a_2, a_3]\to[/mm] X mit
> > [mm]\alpha(p)=:\begin{cases} \delta_1(p), & \mbox{für } p\in[a_1, a_2) \\ \delta_2(p), & \mbox{für } p\in[a_2, a_3]\end{cases}[/mm]
>  
> >  

> > stetig.
>  >  
> > Ist es so offensichtlich, dass [mm]\alpha[/mm] stetig ist, oder
> > sollte ich das noch zeigen?
>
> Man könnte noch kurz begründen, warum [mm]\alpha[/mm] in [mm]p=a_2[/mm]
> stetig ist

Ok, mit linksseitigem und rechtsseitigem Grenzwert ist das leicht einzusehen.

> > Ich habe das ja jetzt nur für 2 "benachbarte" Mengen
> > gezeigt.. ist es denn richtig zu folgern, dass
> > [mm]\bigcup_{k\in\IN}^{}M_k[/mm] wegzusammenhängend ist?
>
> Auch das solltest du noch näher begründen:
>  Durch Wiederholung des gleichen Argumentes (formal durch
> Induktion nach n) folgt, dass [mm]A_n=\bigcup_{k=1}^nM_k[/mm] für
> jedes n wegzusammenhängend ist.
>  Für den Wegzusammenhang der unendlichen Vereinigung
> brauchst du dann dein Argument vom Anfang:
>  Sind zwei Punkte x,y gegeben, so gibt es ein n mit [mm]x,y\in A_n[/mm]
> und damit einen Weg zwischen x und y.

Super, das hört sich doch schon viel präziser an, vielenk Dank!

Grüße, kulli

> > Weil ich
> > am Anfang O.B.d.A geschrieben habe?
>  >  
> > Mfg, kullinarisch
>  


Bezug
Ansicht: [ geschachtelt ] | ^ Forum "Topologie und Geometrie"  | ^^ Alle Foren  | ^ Forenbaum  | Materialien


^ Seitenanfang ^
ev.vorhilfe.de
[ Startseite | Mitglieder | Impressum ]